0 Daumen
263 Aufrufe

blob.png

Text erkannt:

- Beweisen Sie, dass die Ungleichung
$$ \log (x) \leq x-1 $$
für alle \( x \in \mathbb{R}_{>0} \) gilt. Schließen Sie daraus, dass die Ungleichung
$$ \log (x) \leq n\left(x^{1 / n}-1\right) $$
für alle \( x \in \mathbb{R}_{>0} \) und \( n \in \mathbb{N} \) gilt.
- Es sei \( p>1 \). Zeigen Sie, dass
$$ x^{p}+y^{p} \leq(x+y)^{p} \leq 2^{p-1}\left(x^{p}+y^{p}\right) $$
für alle \( x, y \geq 0 \) gilt.

Avatar von

Wie wärs, wenn du für die erste die Ungleichung erstmal als Äquivalenzumformung mit e hoch nimmst und dann die Reihendarstellung Exponentialfunktion verwendest. Wäre meine erste Idee ansonsten hattet ihr bestimmt schon Ungleichung für die Exponentialfunktion.

Ein anderes Problem?

Stell deine Frage

Willkommen bei der Mathelounge! Stell deine Frage einfach und kostenlos

x
Made by a lovely community